Use a double integral to find the volume of the indicated solid

Click For Summary
The discussion focuses on using a double integral to find the volume of a solid, with the initial setup involving the integral from 0 to 2 for both x and y. A participant struggles with the limits of integration, initially setting them incorrectly. After reviewing the graph of the region in the x-y plane, it is clarified that the limits for y should be adjusted to reflect the relationship between x and y. The participant acknowledges the mistake and expresses gratitude for the guidance. The conversation emphasizes the importance of correctly interpreting the limits of integration in double integrals.
iRaid
Messages
558
Reaction score
8

Homework Statement


Use a double integral to find the volume of the indicated solid.
attachment.php?attachmentid=69225&stc=1&d=1398809837.png



Homework Equations





The Attempt at a Solution


I can't find what I did wrong, it seems like a simple problem...
$$\int_0^2 \int_0^x (4-y^{2})dydx=\int_0^2 4x-\frac{x^{3}}{3}dx$$
$$=2x^2-\frac{x^{4}}{12}|_0^2=8-\frac{16}{12}=\frac{20}{3}$$
 

Attachments

  • hw.png
    hw.png
    7.7 KB · Views: 1,576
Physics news on Phys.org
iRaid said:

Homework Statement


Use a double integral to find the volume of the indicated solid.
attachment.php?attachmentid=69225&stc=1&d=1398809837.png



Homework Equations





The Attempt at a Solution


I can't find what I did wrong, it seems like a simple problem...
$$\int_0^2 \int_0^x (4-y^{2})dydx=\int_0^2 4x-\frac{x^{3}}{3}dx$$
$$=2x^2-\frac{x^{4}}{12}|_0^2=8-\frac{16}{12}=\frac{20}{3}$$

One of your limits for ##y## is wrong.
 
Zondrina said:
One of your limits for ##y## is wrong.

I'm not seeing it, sorry.
 
If you graph the region in the x-y plane, it should look something like this:

http://gyazo.com/aedf21fdd2006d58eabea7d5b3324065

Suppose you hold ##x## fixed and allow ##y## to vary. Then clearly from the above graph ##0 ≤ x ≤ 2## and ##x ≤ y ≤ 2##.

Try letting ##y## be fixed and allowing ##x## to vary now. Do you get the same result?
 
  • Like
Likes 1 person
Zondrina said:
If you graph the region in the x-y plane, it should look something like this:

http://gyazo.com/aedf21fdd2006d58eabea7d5b3324065

Suppose you hold ##x## fixed and allow ##y## to vary. Then clearly from the above graph ##0 ≤ x ≤ 2## and ##x ≤ y ≤ 2##.

Try letting ##y## be fixed and allowing ##x## to vary now. Do you get the same result?

Ah right, I feel dumb now.

Thank you.
 
Question: A clock's minute hand has length 4 and its hour hand has length 3. What is the distance between the tips at the moment when it is increasing most rapidly?(Putnam Exam Question) Answer: Making assumption that both the hands moves at constant angular velocities, the answer is ## \sqrt{7} .## But don't you think this assumption is somewhat doubtful and wrong?

Similar threads

  • · Replies 27 ·
Replies
27
Views
3K
  • · Replies 5 ·
Replies
5
Views
2K
Replies
2
Views
2K
  • · Replies 34 ·
2
Replies
34
Views
3K
  • · Replies 10 ·
Replies
10
Views
2K
  • · Replies 1 ·
Replies
1
Views
2K
  • · Replies 3 ·
Replies
3
Views
2K
  • · Replies 4 ·
Replies
4
Views
1K
  • · Replies 9 ·
Replies
9
Views
2K
Replies
12
Views
2K